If exactly five of the stations stay open, which one of the following must close?

Ceci on July 28, 2019

Confused...

I'm confused how I'd make a quick deduction out of this without trying every single answer choice before. Pleased explain deduction!

Replies
Create a free account to read and take part in forum discussions.

Already have an account? log in

Irina on August 7, 2019

@Ceci,

We know from rule (4) that L & R cannot both stay open, so if exactly five stations stay open, and only one is closed, it must be L or R. We can thus eliminate answer choices (B), (C), (D).

Now we know that the rest of the stations are open, and rule (2) tells us that if N stays open, L must stay open. Thus, we can eliminate answer choice (A).

R is thus the one that must close (E)

Does this help?

Let me know if you have any further questions.

mamie on November 2, 2019

Hi Irina, if R is out, how come M is still open?

Rule 3: R -> M
(not R -> not M)

shunhe on January 8, 2020

Hi @mamie,

Thanks for the question. Recall that we can't conclude not R -> not M from R - > M. An example might help. If it rains, I will be wet. But that doesn't mean if it doesn't rain, I won't be wet. Maybe I went swimming or something. Hope this helps.